Most of the cars the Green Pine Auto Dealership sells are minivans and sedans. In January, they sold 10 minivans and 20 sedans. In February, the dealership ran some promotions, and they sold 15 minivans and 25 sedans. During which month did the dealership sell a greater ratio of minivans to sedans?

Answers

Answer 1

Using the concept of ratio, the month of February had a better ratio in sales compared to January

What is Ratio

The ratio is defined as the comparison of two quantities of the same units that indicates how much of one quantity is present in the other quantity.

This is the process of comparing two quantities against one another to determine their ratio against each other.

In the question given;

January = 10 minivans, 20 sedans

The ratio of minivans to sedan = 10 / 20 = 1/2

The ratio of minivans to sedan in January = 1/2

February = 15 minivans, 25 sedans

The ratio of minivans to sedans  = 15/25 = 3/5

The month of February has the highest ratio

Learn more on ratio here;

https://brainly.com/question/2328454

#SPJ1


Related Questions

Find the slope of the line that passes through (5, 9) and (3, -5). Write your answer in simplest form.

Answers

Therefore the slope of the line passes through  (5, 9) and (3, -5) comes out to be 7.

What is slope?

By computing the ratio of the vertical to horizontal distance (rise over run) between two points, the slope—a numerical value that defines how steep a line is—is often calculated.

Here,

The slope of line that passes through  (5, 9) and (3, -5)
Slope= gradient between the two points

Slope= (y2-y1)/(x2-x1)

Where y2= -5

Y1= 9

X2= 3

X1= 5

=> (-5-9)/(3-5)

=> -14/-2

=> 7

Therefore the slope of the line passes through  (5, 9) and (3, -5) comes out to be 7.

To know more about slope , visit

https://brainly.com/question/3605446

#SPJ1

How do you teach regrouping to 2nd graders?

Answers

Answer:Pass out the pennies and the dimes to your students. Write a couple of problems on the board that will use regrouping. As you write them, write “ones” over the ones column. Write “tens” over the tens column. Have students take out their coins for each problem.

Step-by-step explanation:

hese are curves obtained by the intersection of right circular cone and a plane.
a. Parabola
b. Ellipse
c. Hyperbola
d. Conic sections

Answers

Answer: conic sections

Step-by-step explanation: the other options are a part of conic sections that are named depending on the angle of the plane relative to the cone, and the intersection.

5-55.
Additional Challenge: At the beginning of 1990, oil prices were $20 a barrel. Some oil investors predicted that the price
of oil would increase by $2.25 a barrel per year.
In the beginning of 2005, the price of oil was $30 a barrel. With increasing demand for oll around the world, oil investors in
2005 predicted that the price of oil would increase by $5.00 a barrel each year.
DILOIL
a. Let a represent the number of years since 2005. Write an equation that predicts the price of oil, y, using the information available in 2005.
b. Investors in 1990 did not have the benefit of the 2005 information. Write an equation that represents the prediction made in 1990, using the same variables as in
part (a). Remember that a represents the number of years since 2005.
c. Use the equations you wrote in parts (a) and (b) to determine when the cost of a barrel of oil would be the same for both price predictions.
d. In the spring of 2011, a barrel of oil was selling for about $112. Which prediction was closer? Was it a pretty good prediction?

Answers

a) The linear function for the 2005 estimate is given as follows: y = 30 + 5x.

b) The linear function for the 1990 estimate is given as follows: y = 53.75 + 2.25x.

c) They would be predicted to have the same cost in the year of 2013.

d) The 1990 prediction was closer, however it still had a large residual, hence neither was good enough.

How to define the linear functions?

The linear functions are defined in the slope-intercept format, given as follows:

y = mx + b.

In which:

m is the slope, representing the rate of change for the price.b is the y-intercept, representing the initial cost in the reference year.

Taking 2005 as the estimate, the function is given as follows:

y = 30 + 5x.

Taking 1990 as the estimate, the equation would be of:

y = 20 + 2.25x.

The estimate for 2005 would be of:

y = 20 + 2.25(15) = 53.75.

Hence the equation taking 2005 as the estimate would be of:

y = 53.75 + 2.25x.

The costs would be the same when:

30 + 5x = 53.75 + 2.25x.

2.75x = 23.75

x = 23.75/2.75

x = 8.63 -> + 2005 -> year of 2013.

The estimates for 2011(6 years after 2005) are given as follows:

y = 30 + 5x = 30 + 5(6) = $60.y = 53.75 + 2.25(6) = $67.25. (better estimate but not good enough).

More can be learned about linear functions at https://brainly.com/question/24808124

#SPJ1

the area of a rectangle is square feet. determine the length and width if the length is times the width.

Answers

According to the given area of rectangle, the length and width of the rectangle is 7 and 21 respectively.

Area of rectangle

The standard form of calculating the area of the rectangle is written as,

A = l x w

where l refers the length and w refers the width of the rectangle.

Given,

Here we have given that the area of a rectangle is 147 square feet. And we need to determine the length and width if the length is three times the width.

Let us consider length of rectangle is n,

Then the width of rectangle is 3n.

Here we have to remember that the area of a rectangle is

=> A = n x 3n = 3n²

Here we are told that the area,

=> 3n²=147

Then the value of n² = 49

Therefore, the length is 7 and the width is 21.

To know more about Area of rectangle here.

https://brainly.com/question/20693059

#SPJ4

In a simple random sample of 144 households in a city in Kentucky, the average number of children in these households was 1.12 children. The standard deviation from this sample was
2.40 children. A 90 percent confidence interval for the mean number of children in all
households in this city is:
Select one:
O a. 1.12 ± 0.2.
O b. 2.40 ± 0.2.
c. It is impossible to tell without a census.
O d. 1.12 ± 2.40.
O e. 1.12 ± 0.328.

Answers

A 90% confidence interval for the mean number of children in all household in this city is 1.12 ± 0.328, using formula for confidence interval.

What are mean and standard deviation ?

The standard deviation is a summary measure of the differences of each  observation from the mean.

Formula to calculate confidence interval using mean and deviation:

mean ± Z*(std deviation/√n)

where n = sample size,

Given that :

mean = 1.12

deviation = 2.40

n = 144

We know Z is fixed for 90% confidence i.e. 1.64

So, the confidence interval = 1.12 ± 1.64*(2.4/√144)

                                              = 1.12 ± 1.64 * .2

                                               = 1.12 ± .328

Learn more about confidence interval using link :

https://brainly.com/question/14825274

#SPJ1

a popular brand of pen is available in 5 colors and 4 writing tips. how many different choices of pens do you have with this brand?

Answers

If a popular brand of pen is available in 5 colors and 4 writing tips. The  number of different choices of pens do you have with this brand is: 20 choices.

How to find the number of different choices of pen?

Using this formula to find the number of different choices of pen

Number of different choices of pen = Number of  color × Number of writing tips

Where:

Number of color = 5

Number of writing tips = 4

Let plug in the formula

Number of different choices of pen = 5 × 4

Number of different choices of pen = 20 choices

Therefore the number of different choices of pen  is 20 choices.

Learn more about number of different choices of pen here:https://brainly.com/question/16075452

#SPJ1

Order the answers of these following expressions from least to greatest. -13 + 12= . -3 - (-2) + 3= . -3 + 9= . 9 + (-15)= . 13 - (-13)= .

Answers

The answers of the given expressions ordered from least to greatest is

-6, -1, 2, 6, 26

Ordering the answers of expressions from least to greatest

From the question, we are to order the answers of the given expressions from the least to greatest

To do this, we will evaluate the expressions

Evaluating -13 + 12 =

-13 + 12 = -1

Evaluating -3 - (-2) + 3

-3 - (-2) + 3

-3 + 2 + 3 = 2

Evaluating -3 + 9 =

-3 + 9 = 6

Evaluating 9 + (-15)

9 + (-15)

9 - 15 = -6

Evaluating 13 - (-13)

13 - (-13)

13 + 13 = 26

Hence, ordering the answers from least to greatest, we get

-6, -1, 2, 6, 26

Learn more on Ordering the answers of expressions from least to greatest here: https://brainly.com/question/22559078

#SPJ1

For the functions f(t) = 5t and g(t) = sin(t) defined on 0≤t<[infinity], compute f * g in two different ways.a. By directly evaluating the integral in the definition of f * g.b. By computing L−1{F(s)G(s)} where F(s)=L{f(t)} and G(s)=L{g(t)}.

Answers

The laplace inverse transformation L−1{F(s)G(s)} =  5 [tex]\frac{2s}{s^{2} + 1^{2} }[/tex]  

If L{f(t)} = F(s), then the inverse Laplace transform of F(s) is

L−1{F(s)} = f(t). (1)

The inverse transform L−1is a linear operator:

L−1{F(s) + G(s)} = L−1{F(s)} + L−1{G(s)}, (2)

andL−1{cF(s)} = cL−1{F(s)}, (3)

for any constant c

given functions are

f(t) =5t ,g(t) = sin(t)

L(f(t)) = F(s)

L{F(s)*G(s)} = L^-1{5tsin(t)}

   =5L^-1{tsin(t)}

   =5L^-1{[tex]\frac{d}{ds}[/tex] tsin(t)}

    = 5 [tex]\frac{2s}{s^{2} + 1^{2} }[/tex]  

To learn more about inverse Laplace transform :

https://brainly.com/question/1675085

#SPJ4

Which of the following are solutions to the inequality below? Select all that apply.
4>q/23
q = 138
q = 46
q = 69
q = 92

Answers

To get q by itself, we multiply both sides by 23
We get
92>q
q can be 46 or 69 because they are both less than 92

A car that travels 20 miles in 1/2 hour at constant speed is traveling at the same speed as a car that travels 30 miles. In how many hours (at a constant speed)

Answers

The required time to travel 30 miles is given as 45 minutes and 3/4 hours.

Given that,
A car that travels 20 miles in 1/2 hour at constant speed is traveling at the same speed as a car that travels 30 miles.

What is simplification?

The process in mathematics to operate and interpret the function to make the function or expression simple or more understandable is called simplifying and the process is called simplification.

here,
Speed = Distance/time
Speed = 20 / [1/2]
Speed = 40 miles per hour,
Time = Distance / Speed
Time = 30 / 40
Time = 3 / 4 hour

Thus, the required time to travel 30 miles is given as 45 minutes and 3/4 hours.

Learn more about simplification here:

https://brainly.com/question/12501526

#SPJ1

Help me pls pls pls plspl splspls

Answers

I will write the answer in the comments please give me a second

PLEASEEE HELP THIS IS 3 GRAFE LEVELS ABOVE ME!
Check your answer to problem 4 by solving it using a different strategy Show your work.​

Answers

Answer:

you are right

n=1

Step-by-step explanation:

1/2(-3/2n+1)=3/4-n

-3/4n+1/2=3/4-n

-3/4n+2/4=3/4-4/4n

+3/4n               +3/4n

2/4=3/4-1/4n

-3/4. -3/4

-1/4= -1/4n

/-1/4.  /-1/4

1=n

to check i normally just pop in the value (i know no other way in all my years of school)

1/2(-3/2(1)+1)=3/4-1

1/2(-3/2+1=3/4-1

1/2(-1/2)=-1/4

-1/4= -1/4

hopes this helps please mark brainliest

a baseball coach uses a pitching machine to simulate pop flies during practice. the quadratic function f(x)

Answers

The time taken by the baseball in the air if the ball is not caught is 4.513 seconds.

Here we have to find the time taken by the baseball.

Given data:

The quadratic function is given:

f(x) = 16x² + 70x + 10

To find the value of x we have to formula:

x = ±b [tex]\sqrt{b^{2} - 4ac}[/tex] / 2a

from the equation we get the value of a, b, and c as:

a = -16

b = 70

c = -10

now putting in the equation we have:

x = -70 ±[tex]\sqrt{70^{2} -4(-16)(10)}[/tex] / 2(-16)

  = -70 ±[tex]\sqrt{4900 + 640}[/tex] / -32

  = -70 ± √5540 / -32

  = - 70 ± 74.43 / -32

By calculating this we get two values:

x = -0.138 and 4.513

Time can never be negative so.

x = 4.513

Therefore the time taken is 4.513 seconds.

To know more about the quadratic function refer to the link given below:

https://brainly.com/question/1214333

#SPJ4

What are rational numbers but not natural numbers?; Which set of numbers are rational numbers but not integers whole numbers or natural numbers?; Does the set of rational numbers contain the set of natural numbers?; What is a rational number that is not a real number?

Answers

The distinction between them is that an integer is a negative number without a decimal point.

What is integers?

All whole numbers and negative numbers are considered integers. This means if we include negative numbers together with whole numbers, we form a set of integers.

Positive, negative, and zero are all examples of integers.

The Latin word "integer" signifies "whole" or "intact."

As a result, fractions and decimals are not included in integers. In this article, let's learn more about integers, their definition, and their characteristics.

According to our question-

The group of integers that are neither whole nor a natural number are:

As previously noted, the third set of integers lacks a decimal point;

However, the difference between them is that; a negative number without a decimal point is referred to as an integer

From the provided set: only fit into the description of an integer, because

It does not have a decimal point

And that is unfavorable.

learn more about integers click here:

https://brainly.com/question/929808

#SPJ4

Which of the following is an identity?
A. csc²x + cot²x = 1
B. (cscx + cotx)2 = 1
C. sin²x-cos²x = 1
D. sin²x sec²x + 1 = tan²x csc²x

Answers

Answer:

Step-by-step explanation:

it is D as

L.H.S.=sin²x sec²x+1

[tex]\frac{sin^2x}{cos^2x} +1\\=\frac{sin^2x+cos^2x}{cos^2x} \\=\frac{1}{cos^2x} \\=sec^2x[/tex]

again

[tex]R.H.S.=tan^2x csc^2x\\=\frac{sin^2x}{cos^2x} \times \frac{1}{sin^2x} \\=\frac{1}{cos^2x} \\=sec^2x[/tex]

L.H.S.=R.H.S.

which option proves the following statement by contradiction? for all real numbers r and s, if r is rational and s is irrational, then r 2s is irrational. proof (by contradiction): suppose not. that is, suppose there are real numbers r and s such that r is rational and s is irrational and r 2s is rational. [we must show that this supposition leads logically to a contradiction.] by definition of rational,

Answers

Any whole number, fraction, or decimal that terminates or repeats is a rational number. Any number that cannot be divided into a fraction and hence does not meet the concept of a rational number is said to be irrational.

When it comes to rationality, we have

r + 2s = c/d and r = a/b.

The result of the substitution is a/b + 2s = c/d

2s= c/d-a/b=(bc-ad)/bd

s = (bc-ad)/2bd

Now that both (bc - ad) and (bd) are integers, bd is not equal to 0

As a result, the integer s is a quotient of the two integers (bc - ad) where bd is not equal to 0. So, s is rational according to the definition. This disproves the idea that s is irrational.

Want to know more about rational numbers visit the link which is given below;

https://brainly.com/question/24398433

#SPJ4

The domestic price of shoes is $80. After trade the price of a pair of shoes is $60. After trade this country will import

Answers

The trade this country will import is given by the option (C) 300 pair of shoes.

The number of shoes exported will be calculated by the formula -

Import = quantity demanded - quantity supplied. The formula is based on the concept that the import of shoes will be carried out once the manufacturing is more than the demand.

The graph indicates the quantity demanded is 1000 and the quantity supplied is 1300. Keep the values in formula to find imports.

Imports = 1300 - 1000

Performing subtraction on Right Hand Side of the equation

Imports = 300

Hence, the import is 300.

Learn more about import -

https://brainly.com/question/24473707

#SPJ4

The complete question is attached in figure.

To evaluate forecasting performance of a model or compare relative performance of two
models, examine
a. contingency forecasts
b. conditional forecasts
c. ex ante forecasts
d. ex post forecasts
e. all of these

Answers

Therefore, the answer to this question is option (C) ex ante forecasts

What is  forecasting performance?

To face the challenge of improving operational performance while increasing production efficiency, performance forecasting is a crucial service to help decision-making in the idea, design, and operational phases of an asset.

Here,

As to evaluate forecasting performance of a model or compare relative performance of two models,

can be examined on the basis of ex ante forecasts

The term "ex-ante forecast" refers to a forecast that only takes into account the data (i.e., the value of economic variables) that is available at the time of the actual forecast and not the value of some variables that will be available later on.

Therefore, the answer to this question is option (C) ex ante forecasts

To know more about forecast , visit

https://brainly.com/question/27994086

#SPJ4

a hypothesis test was performed to decide whether houses are more prone to being lost to fire than condominiums are. the researcher looked at 1500 randomly selected houses and 1200 randomly selected condominiums over a ten year period. then, since over 30 of each were in the sample, the researcher can use a z-test for the difference between two proportions.
o False
o True

Answers

It is false, the researcher cannot use a z-test for the difference between two proportions.

A hypothesis test was performed to decide whether houses are more prone to being lost to fire than condominiums are. the researcher looked at 1500 randomly selected houses and 1200 randomly selected condominiums over a ten year period. then, since over 30 of each were in the sample, the researcher cannot use a z-test for the difference between two proportions.

z = (X - mean)/(SD/√n)

Where SD is the standard deviation and n is the number of observations.

Here, the required value is not provided. So, we cannot use the z - test.

Therefore, the researcher cannot use a z-test for the difference between two proportions, so the given statement is false.

To learn more about z test refer here

https://brainly.com/question/14453510

#SPJ4

At the time of her​ grandson's birth, a grandmother deposits 14000 in an account that pays 9.5% compounded monthly. What will be the value of the account at the​ child's twenty-first​ birthday, assuming that no other deposits or withdrawals are made during this​ period?

Answers

The value of the account at the​ child's twenty-first​ birthday is 102125.44

How to determine the value of the account at the​ child's twenty-first​ birthday?

We can use the compound interest formula to determine the value of the account (A) on the​ child's twenty-first​ birthday:

A = P (1 + r/n)^(nt)

Where:

P is the initial amount of money deposited in the account

r is the interest rate

n is the number of times the interest is compounded per year

t is the number of years

Given: P = 14000, r = 9.5% = 0.095, t = 21, n = 12 (i.e. 12 months per year)

Substitututing into A = P (1 + r/n)^(nt):

A = 14000 (1 +  0.095/12)^(12×21)

A = 14000 (1 +  0.095/12)^(252)

A = 102125.44

Learn more about compound interest on:

https://brainly.com/question/24274034

#SPJ1

Can someone help me with this, please?

Answers

π√π÷√π  is an irrational number. So option (a) is required answer.

What is irrational number?

A real number that cannot be stated as a ratio of integers is said to be irrational; an example of this is the number 2. Any irrational number, such as p/q, where p and q are integers, q, cannot be expressed as a ratio. Once more, an irrational number's decimal expansion is neither ending nor recurrent.

Examples of irrational number: √3, ∛2, π

π is an irrational because it is neither ending nor recurrent.

Solving each part of options, we get

π√π÷√π = π

√25 = 5

√π÷π = 1

2√2 ÷ √2 = 2

As most of the option give rational number only one option gives irrational number which is π

We get π from option (a)

So, option (a) is correct option.

Learn more about irrational number here:

https://brainly.com/question/43641

#SPJ1

The diameter of a quarter is about 1 in.
You trace around the edge of the quarter on a sheet of paper.
What is the area of the circle on the paper?

Use 3.14 as an approximation for π. Round your answer to the nearest tenth.

Answers

Answer: The area is going to be 0.8

Step-by-step explanation:  The diameter is 1

You need the radius of the circle which is half the diameter. So the Radius(R) is 0.5.

After you find the radius of the circle to find the area of the circle you use the formula pie (R)^2. You are using 3.14

3.14(0.5)^2= 0.785

you then want to round your answer to the nearest tenth so the 8 turns the 7 into an 8.

So, the Area is 0.785 but after rounding it is 0.8.

Hope that helped :)

$6,000 for six years at 8½% compounded daily will grow to:
Multiple Choice
$9,060.00
$9,788.81
$9,991.15

Answers

It will grow to $9991.15

How to get
The midpoint of GH is M(-6,-3). One endpoint is H (-4,4). Find the coordinates of endpoint G

Answers

The coordinates of point G are (-8, -10)

What is the midpoint of a line?

A midpoint of a line segment is the point on a segment that bisects the segment into two congruent segments

The midpoint of a line segment is the point on a segment that is at the same distance or halfway between the two ending points.

midpoint(M) = (-6, -3)

H = (-4, 4)

let the coordinates of G be (p,q)

taking the x coordinates for M, G and H

-6 = p -4/2

cross multiply

p -4 = -12

p = -12 + 4

p = -8

taking y-coordinates

-3 = q +4/2

q + 4 = -6

q = -6 -4

q = -10

coordinates of G (-8, -10)

in conclusion the coordinates of G is ( -8, -10)

Learn more about midpoint of line: https://brainly.com/question/18315903

#SPJ1

• For which of the following equations is (3, - 1) not a solution?

Answers

Therefore , x +3 =y  is the equation which is not the solution of point (3,-1)

What is equation ?

Equations are logical assertions in mathematics that have two algebraic expressions on either side of an equals (=) sign. It is demonstrated that the expressions on the left and right are equivalent to one another. LHS = RHS (left hand side = right hand side) appears in all mathematical equations.

Here,

The given equations are ( a) x-4= y (b) 2x -7 =y (c ) x +3 =y ( d) x/3 = -y

Thus from the given equation

We put, values in each of equation to find that values satisfy the equation or not

Thus, we found  x +3 =y does not satisfy the point (3,-1)

Therefore , x +3 =y  is the equation which is not the solution of point (3,-1)

To know more about equation , visit

https://brainly.com/question/12788590

#SPJ1

The correct question is -

For which of the following equations is (3, - 1) not a solution?

Group of answer choices -      y=x-1, -3x=4y-6, 2y-3x=0, 5x+2y=-16

Each person in a random sample of adults was asked how many DVD, he or she wwned. Summary Statistics are given below Variable DVDs N 117 Mean 129.4 Median 50.0 TIMean 76.5 SE Mean StDev 3236 Variable DVDs Minimum 00 Maximum 3000.0 Q1 30.0 03 950 Which of the following statements is true? (A) Seventy-five percent of the adults in the sample own more than 95 DVDs. (B) Fifty percent of the adults in the sample own between 0 and 129.4 DVDs (C) The distribution of the number of DVDs owned appears to be approximately symmetric (D) The interquartile range of the number of DVDs owned is 65. (E) The distribution of the number of DVDs owned contains outliers on both the low side and the high side.

Answers

Hence, the option D "The interquartile range of the number of DVDs owned is 65" is correct.

In the given question, we have to in each person in a random sample of adults was asked how many DVD, he or she owned.

Summary Statistics are given below

Variable     N     Mean     Median     TiMean     St Dev     St Mean

DVDs         117    129.4       50.0          76.5           3236           29.2

Variable     Minimum        Maximum                    Q1            Q3

DVDs              00                3000                         30.0          95.0

We have to which of the following statements is true?

(A) Seventy-five percent of the adults in the sample own more than 95 DVDs.

(B) Fifty percent of the adults in the sample own between 0 and 129.4 DVDs

(C) The distribution of the number of DVDs owned appears to be approximately symmetric

(D) The interquartile range of the number of DVDs owned is 65.

(E) The distribution of the number of DVDs owned contains outliers on both the low side and the high side.

Option (A) is incorrect because 95 is third quartile value and there is only 25% values above third quartile. So, statement is incorrect.

Option (B) is incorrect because 0 is minimum value and 129.4 is mean value. Percent between mean and minimum is not always fixed, so we cannot say that 50% of values are between 0 and 129.4. So, statement is incorrect.

Option (C) is incorrect because median and mean must be equal for symmetric curve, but these are not equal. So, it is not a symmetric curve.

Option (D) is correct because

IQR (Interquartile range)

IQR = Q3 - Q1

IQR = 95 - 30

IQR = 65.

So, option D is correct answer.

Option (E) is incorrect because

lower outlier limit = Q1-1.5*IQR

lower outlier limit = 30 - 1.5*65

lower outlier limit = -67.5

upper outlier limit = Q3 + 1.5*IQR

upper outlier limit = 95 + 1.5*65

upper outlier limit = 192.5

So, there is outliers only on upper side because maximum is 3000>192.5, but there is not value below 0.

To learn more about mean, median and range link is here

brainly.com/question/9026799

#SPJ4

for each of the three plots, identify the strength of the relationship (e.g. weak, moderate, or strong) in the data and whether fitting a linear model would be reasonable.

Answers

These following answers are correct

We are given residual plots

In given(a) plot there is weak relationship between scatter points

we see curved relationship in this residual plot therefore model is unreasonable .

Note - This model is reasonable when this plot should look approximately normal and scatterplot of residuals should show random

scatter. In these plots we see weak relationship and line as model is unreasonable

2)We are given residual plot.

Given 2nd (c) plot there is strong relationship between scatter points we see positive straigh line relationship in this plot therefore model is resonable.

Also these model look approximately normal land scatterplot of residuals show random scatter.

In these plot we see strong relationship and linear model is reasonable

Therefore, these statements are correct.

To learn more about scatterplot click here:

brainly.com/question/29366075

#SPJ4

What is ? x 5 = 9 for 5th grade

Answers

Answer:

The answer is 1.8

Step-by-step explanation:

Let the number be y

So,

y × 5 = 9

5y = 9

Divide both side by 5

5y/5 = 9/5

y = 9/5 = 1.8

Thus, The value of y is 1.8

Which statement makes an accurate comparison of the motions for objects C and D?

Answers

Derivatives are used to determine that the following is the best statement:

The acceleration of object C is greater than the acceleration of object D.

When you divide the change in velocity by the change in time, you get the acceleration, which is: a=Δv/Δt

In this issue:

Both object C and object T experience an identical shift in time.The more rapid change in velocity for object C causes it to accelerate more quickly.

hence the right answer is:

The acceleration of object C is greater than the acceleration of object D.

Learn more about on comparison, here:

https://brainly.com/question/9107349

#SPJ4

Answer:

Object C had an acceleration that is greater than the acceleration for D

Step-by-step explanation:

Other Questions
which statements describe the poem select three options which one of the following is the focus of activity-based management? a. to improve allocation of indirect production costs. b. to reduce the number of cost pools. c. to increase the number of volume-related allocation bases. d. to improve the effectiveness of activities. desk costs $195 in the united states. the same desk costs 150 british pounds. if purchasing power parity holds perfectly, what is the exchange rate in dollars per pound quzilet Which of these statements from the passage indicates the authors bias toward volunteer efforts of young people? match each muscle with its corresponding motor innervation. each term can be used more than once. be as specific as possible. The wholesale price for a shirt is 5.50. A certain department store marks up the wholesale price by 90%. Find the price of the shirt in the department store.Round your answer to the nearest cent, as necessary. Two mobile phone companies have different data rates. Company A has a rate of$100 for unlimited data, and Company B has a flat fee of $20 plus $5 per gigabyte of data. The point of intersection between these two linear relations is (16, 100).What does this point of intersection represent?a) Both phone companies would charge $16 per gigabyte used with a bill of $100.b) Both phone companies would charge $100 if the customer used 16 gigabytes of data.c) Both companies would charge $100 with a flat fee of $16.d) Both companies would charge $16 per gigabyte if the customer used 100 gigabytes. The Product Manager is responsible for which three activities? (Choose three.)Establishing Features and Benefit HypothesisAssigning Agile Release Trains to a common technological and architectural visionDeveloping user story acceptance criteriaDefining Program Backlog contentPrioritizing Features for optimum economic valuePrioritizing Capabilities and Enablers A bag contains 6 red tiles, 8 blue tiles, and 6 green tiles. A tile is drawn , the color recorded, and then the tile placed back in the bag before a tile is drawn again. If this procedure is done 30 times, how many red tiles would you expect to draw?Responses of a certain Compound X, known to be made of carbon, hydrogen and perhaps oxygen, and to have a molecular molar mass of 128./gmol, is burned completely in excess oxygen, and the mass of the products carefully measured: product mass carbon dioxide 22.35g water 3.66g Use this information to find the molecular formula of X. Simplify the expression. negative 17 plus the quantity negative 3 and seven tenths plus 9 and 15 hundredths end quantity divided by 5 all times 4 squared minus 4 and 7 tenths 4.26 12.98 13.41 259.26 Which of the following is a polynomial ? (a) 1/x+1 (b) 1/x+1 (c) 1/x+x+1 (d) x+1 a 0.341-g wire is stretched between two points 70.5 cm apart. if the tension in the wire is 578 n, find the frequencies of the wire's (a) first, (b) second, and (c) third harmonics. for the first harmonic, the length of the wire is a half wavelength. The variable p varies inversely as the square of q. When p = 36, q = 25.When p = 4, q=When q = 10, p = The economy begins in long-run equilibrium. Then one day, the president appoints a new chairman of the Fed. This new chairman is well known for her view that inflation is not a major problem for any economy.1. How would this news affect the price level that people would expect to prevail? Explain2. How would this change affect the nominal wage that workers and firms agree to in their new labor contracts? Explain3. How would this change affect the profitability of producing goods and services at any given price level? Explain. What is the medianof 98, 99, 100, 90,98, 100? researchers recorded that a certain bacteria population declined from 270,000 to 200 in 36 hours. at this rate of decay, how many bacteria will there be in 18 hours? round to the nearest whole number what is the smallest one-dimensional box in which you can confine an electron if you want to know for certain that the electron's speed is no more than 10 m/sm/s ? the coefficients of dummy variables in multiple regression models must always be positive. group of answer choices true false complete the valence molecular orbital diagram for oxygen o2